Monday 15 May 2017

inequality - Lower and upper bound of the Stirling's approximation

Perhaps everybody has heard of the Stirling's approximation, namely:
$$ \Gamma(z)\approx\sqrt{\frac{2\pi}{z}}\left(\frac{z}{e}\right)^z $$
Thus (the very basic example):
$$ \Gamma\left(\frac{1}{2}z\right) \approx\sqrt{\frac{4\pi}{z}}\left(\sqrt{\frac{z}{2e}}\right)^{z}$$
My question is: how does one obtain the lower and upper bound for the Gamma function using the Stirling's approximation? I've heard that $$ \sqrt{\frac{4\pi}{z}}\left(\sqrt{\frac{z}{2e}}\right)^{\color{red}{z-1}}<\Gamma\left(\frac{1}{2}z\right)<\sqrt{\frac{4\pi}{z}}\left(\sqrt{\frac{z}{2e}}\right)^{\color{red}{z+1}} $$ is a very "ugly" and unproper way (moreover it doesn't always work). So what's the best way to obtain the lower and upper bound?

No comments:

Post a Comment

real analysis - How to find $lim_{hrightarrow 0}frac{sin(ha)}{h}$

How to find $\lim_{h\rightarrow 0}\frac{\sin(ha)}{h}$ without lhopital rule? I know when I use lhopital I easy get $$ \lim_{h\rightarrow 0}...